How do I find the convolution of two functions with different domains?

Click For Summary
To find the convolution of the functions β and L, the integral must be set up correctly, considering the limits of integration based on the domains of the functions. The region of integration is defined by the intersection of the support of β and L, which varies depending on the value of a in relation to 1. For values of x, the integral will either be zero or will have limits defined by the intersection points of the two functions. Understanding these limits is crucial for correctly calculating the convolution. The discussion emphasizes the importance of visualizing the integration region to determine the appropriate bounds for the integral.
bobred
Messages
170
Reaction score
0

Homework Statement


I have the two functions below and have to find the convolution \beta * L

Homework Equations


Assume a<1
<br /> \beta(x)=\begin{cases}<br /> \frac{\pi}{4a}\cos\left(\frac{\pi x}{2a}\right) &amp; \left|x\right|&lt;a\\<br /> 0 &amp; \left|x\right|\geq a<br /> \end{cases}<br />
<br /> L(x)=\begin{cases}<br /> 1 &amp; \left|x\right|&lt;1\\<br /> 0 &amp; \left|x\right|\geq 1<br /> \end{cases}<br />

The Attempt at a Solution


\beta * L=\int^\infty_{-\infty}\beta(y)L(x-y)dy=\int^\infty_{-\infty}\beta(x-y)L(y)dy
What I am unsure about is the limits and which integral to use. Any hints would be greatly appreciated.
 
Physics news on Phys.org
bobred said:

Homework Statement


I have the two functions below and have to find the convolution \beta * L

Homework Equations


Assume a<1
<br /> \beta(x)=\begin{cases}<br /> \frac{\pi}{4a}\cos\left(\frac{\pi x}{2a}\right) &amp; \left|x\right|&lt;a\\<br /> 0 &amp; \left|x\right|\geq a<br /> \end{cases}<br />
<br /> L(x)=\begin{cases}<br /> 1 &amp; \left|x\right|&lt;1\\<br /> 0 &amp; \left|x\right|\geq 1<br /> \end{cases}<br />

The Attempt at a Solution


\beta * L=\int^\infty_{-\infty}\beta(y)L(x-y)dy=\int^\infty_{-\infty}\beta(x-y)L(y)dy
What I am unsure about is the limits and which integral to use. Any hints would be greatly appreciated.

Start by drawing the two-dimensional ##A## region for the second form:
$$A_a = \{ (x,y) : -1 \leq y \leq 1, |x-y| \leq a \}.$$
There are three cases: (1) ##a < 1##; (2) ##a = 1##; and (3) ##a > 1##. The region will look slightly different in cases (1) and (3).For some values of ##x## the line vertical line through ##x## will miss region ##A_a## completely, so the ##y##integral will equal 0. For other values of ##x## the vertical line through ##x## will intersect region ##A_a## in a vertical line segment, so will have the form ##y_1(x) \leq y \leq y_2(x)##, and your ##y##-integration will be from ##y_1(x)## to ##y_2(x)##.
 
Question: A clock's minute hand has length 4 and its hour hand has length 3. What is the distance between the tips at the moment when it is increasing most rapidly?(Putnam Exam Question) Answer: Making assumption that both the hands moves at constant angular velocities, the answer is ## \sqrt{7} .## But don't you think this assumption is somewhat doubtful and wrong?

Similar threads

  • · Replies 10 ·
Replies
10
Views
2K
  • · Replies 11 ·
Replies
11
Views
3K
  • · Replies 6 ·
Replies
6
Views
1K
  • · Replies 5 ·
Replies
5
Views
2K
Replies
8
Views
2K
  • · Replies 4 ·
Replies
4
Views
2K
  • · Replies 19 ·
Replies
19
Views
29K
  • · Replies 4 ·
Replies
4
Views
1K
Replies
4
Views
2K
  • · Replies 17 ·
Replies
17
Views
5K